How to prove this inequality involving the minimum of two chosen numbers

The name of the pictureThe name of the pictureThe name of the pictureClash Royale CLAN TAG#URR8PPP











up vote
11
down vote

favorite
5












Let $a_1, b_1, a_2, b_2, dots , a_n, b_n$ be nonnegative real numbers. Prove that



$$sum_i, j = 1^n mina_ia_j, b_ib_j le sum_i, j = 1^n mina_ib_j, a_jb_i.$$



How do I prove this inequality. I have studied only basic inequalities, namely AM-GM, CS inequality and Tchebycheff's inequality. How do I incorporate that condition of minimum of the two chosen numbers and prove the inequality using the above ones
?










share|cite|improve this question



























    up vote
    11
    down vote

    favorite
    5












    Let $a_1, b_1, a_2, b_2, dots , a_n, b_n$ be nonnegative real numbers. Prove that



    $$sum_i, j = 1^n mina_ia_j, b_ib_j le sum_i, j = 1^n mina_ib_j, a_jb_i.$$



    How do I prove this inequality. I have studied only basic inequalities, namely AM-GM, CS inequality and Tchebycheff's inequality. How do I incorporate that condition of minimum of the two chosen numbers and prove the inequality using the above ones
    ?










    share|cite|improve this question

























      up vote
      11
      down vote

      favorite
      5









      up vote
      11
      down vote

      favorite
      5






      5





      Let $a_1, b_1, a_2, b_2, dots , a_n, b_n$ be nonnegative real numbers. Prove that



      $$sum_i, j = 1^n mina_ia_j, b_ib_j le sum_i, j = 1^n mina_ib_j, a_jb_i.$$



      How do I prove this inequality. I have studied only basic inequalities, namely AM-GM, CS inequality and Tchebycheff's inequality. How do I incorporate that condition of minimum of the two chosen numbers and prove the inequality using the above ones
      ?










      share|cite|improve this question















      Let $a_1, b_1, a_2, b_2, dots , a_n, b_n$ be nonnegative real numbers. Prove that



      $$sum_i, j = 1^n mina_ia_j, b_ib_j le sum_i, j = 1^n mina_ib_j, a_jb_i.$$



      How do I prove this inequality. I have studied only basic inequalities, namely AM-GM, CS inequality and Tchebycheff's inequality. How do I incorporate that condition of minimum of the two chosen numbers and prove the inequality using the above ones
      ?







      inequality summation contest-math maxima-minima substitution






      share|cite|improve this question















      share|cite|improve this question













      share|cite|improve this question




      share|cite|improve this question








      edited Sep 10 at 20:51









      Michael Rozenberg

      89.3k1583179




      89.3k1583179










      asked Sep 10 at 13:32









      saisanjeev

      555212




      555212




















          1 Answer
          1






          active

          oldest

          votes

















          up vote
          5
          down vote













          It's USA 2000. The following solution is not mine.



          Since if $mina_i,b_i=0$ then $mina_ia_j,b_ib_j=mina_ib_j,a_jb_i=0,$



          we can assume that $a_i>0$ and $b_i>0$.



          Lemma 1.



          Let $r_igeq0$ and $x_i$ be real numbers. Prove that:
          $$sum_i,j=1^nminr_i,r_jx_ix_jgeq0.$$
          Proof.



          Since $minr_i,r_j=minr_j,r_i,$ we can assume that $0=r_0leq r_1leq r_2leq...leq r_n$ and we obtain:
          $$sum_i,j=1^nminr_i,r_jx_ix_j=sum_i=1^nr_ix_i^2+2sum_1leq i<jleq nr_ix_ix_j=sum_i=1^nleft(r_i-r_i-1right)left(sum_j=i^nx_jright)^2geq0.$$
          Lemma 2.



          Let $a_i>0$, $b_i>0$, $r_i=fracmaxa_i,b_imina_i,b_i-1$ and $x_i=sign(a_i-b_i)mina_i,b_i.$ Prove that:
          $$mina_ib_j,a_jb_i-mina_ia_j,b_ib_j=minr_i,r_jx_ix_j.$$
          Proof.



          Since replacing $a_i$ with $b_i$ gives replacing signs of the both sides, we can assume that $a_igeq b_i$.



          Similarly, we can assume that $a_jgeq b_j$ and we obtain:
          $$minr_i,r_jx_ix_j=minleftfraca_ib_i-1,fraca_jb_j-1rightb_ib_j=$$
          $$=mina_ib_j-b_ib_j,a_jb_i-b_ib_j=mina_ib_j,a_jb_i-b_ib_j=$$
          $$=mina_ib_j,a_jb_i-mina_ia_j,b_ib_j.$$
          Now, by applying of these lemmas we obtain:
          $$sum_i,j=1^nmina_ib_j,a_jb_i-sum_i,j=1^nmina_ia_j,b_ib_j=$$
          $$=sum_i,j=1^nleft(mina_ib_j,a_jb_i-mina_ia_j,b_ib_jright)=sum_i,j=1^nminr_i,r_jx_ix_jgeq0$$ and we are done!



          For $n=4$ we obtain:
          $$(r_4-r_3)x_4^2+(r_3-r_2)(x_3+x_4)^2+(r_2-r_1)(x_2+x_3+x_4)^2+(r_1-r_0)(x_1+x_2+x_3+x_4)^2=$$
          $$=r_1x_1^2+(r_2-r_1+r_1)x_2^2+(r_3-r_2+r_2-r_1+r_1)x_3^2+(r_4-r_3+r_3-r_2+r_2-r_1+r_1)x_4^2+$$
          $$+2r_1x_1x_2+2r_1x_1x_3+2r_1x_1x_4+2(r_2-r_1+r_1)x_2x_3+$$
          $$+2(r_2-r_1+r_1)x_2x_4+2(r_3-r_2+r_2-r_1+r_1)x_3x_4=$$
          $$=r_1x_1^2+r_2x_2^2+x_3r_3^2+r_4x_4^2+2r_1x_1x_2+2r_1x_1x_3+2r_1x_1x_4+$$
          $$+2r_2x_2x_3+2r_2x_2x_4+2r_3x_3x_4.$$






          share|cite|improve this answer






















          • Have you used any telescopic property of series to achieve equality $sum_i=1^nr_ix_i^2+2sum_1leq i<jleq nr_ix_ix_j=sum_i=1^nleft(r_i-r_i-1right)left(sum_j=i^nx_jright)^2$?
            – MathOverview
            Sep 10 at 20:56










          • @MathOverview It's just telescoping summation and $left(sumlimits_i=1^kx_iright)^2=sumlimits_i=1^kx_i^2+2sumlimits_1leq i<jleq kx_ix_j.$
            – Michael Rozenberg
            Sep 10 at 20:58











          • I know that telescopic property for series was somehow used. But I can not understand the equality between these two quantities. Could you explain better?
            – MathOverview
            Sep 10 at 20:59










          • @MathOverview For example, the coefficient before $x_k^2$ it's $sumlimits_i=1^k(r_i-r_i-1)=r_k-r_0=r_k.$ Similarly, the coefficient before $x_ix_j$, where $i<j$ it's $2r_i$.
            – Michael Rozenberg
            Sep 10 at 21:07











          • @MichaelRozenberg can you please elaborate on the step pointed out earlier by MathOverview
            – saisanjeev
            Sep 11 at 14:08










          Your Answer




          StackExchange.ifUsing("editor", function ()
          return StackExchange.using("mathjaxEditing", function ()
          StackExchange.MarkdownEditor.creationCallbacks.add(function (editor, postfix)
          StackExchange.mathjaxEditing.prepareWmdForMathJax(editor, postfix, [["$", "$"], ["\\(","\\)"]]);
          );
          );
          , "mathjax-editing");

          StackExchange.ready(function()
          var channelOptions =
          tags: "".split(" "),
          id: "69"
          ;
          initTagRenderer("".split(" "), "".split(" "), channelOptions);

          StackExchange.using("externalEditor", function()
          // Have to fire editor after snippets, if snippets enabled
          if (StackExchange.settings.snippets.snippetsEnabled)
          StackExchange.using("snippets", function()
          createEditor();
          );

          else
          createEditor();

          );

          function createEditor()
          StackExchange.prepareEditor(
          heartbeatType: 'answer',
          convertImagesToLinks: true,
          noModals: false,
          showLowRepImageUploadWarning: true,
          reputationToPostImages: 10,
          bindNavPrevention: true,
          postfix: "",
          noCode: true, onDemand: true,
          discardSelector: ".discard-answer"
          ,immediatelyShowMarkdownHelp:true
          );



          );













           

          draft saved


          draft discarded


















          StackExchange.ready(
          function ()
          StackExchange.openid.initPostLogin('.new-post-login', 'https%3a%2f%2fmath.stackexchange.com%2fquestions%2f2911918%2fhow-to-prove-this-inequality-involving-the-minimum-of-two-chosen-numbers%23new-answer', 'question_page');

          );

          Post as a guest






























          1 Answer
          1






          active

          oldest

          votes








          1 Answer
          1






          active

          oldest

          votes









          active

          oldest

          votes






          active

          oldest

          votes








          up vote
          5
          down vote













          It's USA 2000. The following solution is not mine.



          Since if $mina_i,b_i=0$ then $mina_ia_j,b_ib_j=mina_ib_j,a_jb_i=0,$



          we can assume that $a_i>0$ and $b_i>0$.



          Lemma 1.



          Let $r_igeq0$ and $x_i$ be real numbers. Prove that:
          $$sum_i,j=1^nminr_i,r_jx_ix_jgeq0.$$
          Proof.



          Since $minr_i,r_j=minr_j,r_i,$ we can assume that $0=r_0leq r_1leq r_2leq...leq r_n$ and we obtain:
          $$sum_i,j=1^nminr_i,r_jx_ix_j=sum_i=1^nr_ix_i^2+2sum_1leq i<jleq nr_ix_ix_j=sum_i=1^nleft(r_i-r_i-1right)left(sum_j=i^nx_jright)^2geq0.$$
          Lemma 2.



          Let $a_i>0$, $b_i>0$, $r_i=fracmaxa_i,b_imina_i,b_i-1$ and $x_i=sign(a_i-b_i)mina_i,b_i.$ Prove that:
          $$mina_ib_j,a_jb_i-mina_ia_j,b_ib_j=minr_i,r_jx_ix_j.$$
          Proof.



          Since replacing $a_i$ with $b_i$ gives replacing signs of the both sides, we can assume that $a_igeq b_i$.



          Similarly, we can assume that $a_jgeq b_j$ and we obtain:
          $$minr_i,r_jx_ix_j=minleftfraca_ib_i-1,fraca_jb_j-1rightb_ib_j=$$
          $$=mina_ib_j-b_ib_j,a_jb_i-b_ib_j=mina_ib_j,a_jb_i-b_ib_j=$$
          $$=mina_ib_j,a_jb_i-mina_ia_j,b_ib_j.$$
          Now, by applying of these lemmas we obtain:
          $$sum_i,j=1^nmina_ib_j,a_jb_i-sum_i,j=1^nmina_ia_j,b_ib_j=$$
          $$=sum_i,j=1^nleft(mina_ib_j,a_jb_i-mina_ia_j,b_ib_jright)=sum_i,j=1^nminr_i,r_jx_ix_jgeq0$$ and we are done!



          For $n=4$ we obtain:
          $$(r_4-r_3)x_4^2+(r_3-r_2)(x_3+x_4)^2+(r_2-r_1)(x_2+x_3+x_4)^2+(r_1-r_0)(x_1+x_2+x_3+x_4)^2=$$
          $$=r_1x_1^2+(r_2-r_1+r_1)x_2^2+(r_3-r_2+r_2-r_1+r_1)x_3^2+(r_4-r_3+r_3-r_2+r_2-r_1+r_1)x_4^2+$$
          $$+2r_1x_1x_2+2r_1x_1x_3+2r_1x_1x_4+2(r_2-r_1+r_1)x_2x_3+$$
          $$+2(r_2-r_1+r_1)x_2x_4+2(r_3-r_2+r_2-r_1+r_1)x_3x_4=$$
          $$=r_1x_1^2+r_2x_2^2+x_3r_3^2+r_4x_4^2+2r_1x_1x_2+2r_1x_1x_3+2r_1x_1x_4+$$
          $$+2r_2x_2x_3+2r_2x_2x_4+2r_3x_3x_4.$$






          share|cite|improve this answer






















          • Have you used any telescopic property of series to achieve equality $sum_i=1^nr_ix_i^2+2sum_1leq i<jleq nr_ix_ix_j=sum_i=1^nleft(r_i-r_i-1right)left(sum_j=i^nx_jright)^2$?
            – MathOverview
            Sep 10 at 20:56










          • @MathOverview It's just telescoping summation and $left(sumlimits_i=1^kx_iright)^2=sumlimits_i=1^kx_i^2+2sumlimits_1leq i<jleq kx_ix_j.$
            – Michael Rozenberg
            Sep 10 at 20:58











          • I know that telescopic property for series was somehow used. But I can not understand the equality between these two quantities. Could you explain better?
            – MathOverview
            Sep 10 at 20:59










          • @MathOverview For example, the coefficient before $x_k^2$ it's $sumlimits_i=1^k(r_i-r_i-1)=r_k-r_0=r_k.$ Similarly, the coefficient before $x_ix_j$, where $i<j$ it's $2r_i$.
            – Michael Rozenberg
            Sep 10 at 21:07











          • @MichaelRozenberg can you please elaborate on the step pointed out earlier by MathOverview
            – saisanjeev
            Sep 11 at 14:08














          up vote
          5
          down vote













          It's USA 2000. The following solution is not mine.



          Since if $mina_i,b_i=0$ then $mina_ia_j,b_ib_j=mina_ib_j,a_jb_i=0,$



          we can assume that $a_i>0$ and $b_i>0$.



          Lemma 1.



          Let $r_igeq0$ and $x_i$ be real numbers. Prove that:
          $$sum_i,j=1^nminr_i,r_jx_ix_jgeq0.$$
          Proof.



          Since $minr_i,r_j=minr_j,r_i,$ we can assume that $0=r_0leq r_1leq r_2leq...leq r_n$ and we obtain:
          $$sum_i,j=1^nminr_i,r_jx_ix_j=sum_i=1^nr_ix_i^2+2sum_1leq i<jleq nr_ix_ix_j=sum_i=1^nleft(r_i-r_i-1right)left(sum_j=i^nx_jright)^2geq0.$$
          Lemma 2.



          Let $a_i>0$, $b_i>0$, $r_i=fracmaxa_i,b_imina_i,b_i-1$ and $x_i=sign(a_i-b_i)mina_i,b_i.$ Prove that:
          $$mina_ib_j,a_jb_i-mina_ia_j,b_ib_j=minr_i,r_jx_ix_j.$$
          Proof.



          Since replacing $a_i$ with $b_i$ gives replacing signs of the both sides, we can assume that $a_igeq b_i$.



          Similarly, we can assume that $a_jgeq b_j$ and we obtain:
          $$minr_i,r_jx_ix_j=minleftfraca_ib_i-1,fraca_jb_j-1rightb_ib_j=$$
          $$=mina_ib_j-b_ib_j,a_jb_i-b_ib_j=mina_ib_j,a_jb_i-b_ib_j=$$
          $$=mina_ib_j,a_jb_i-mina_ia_j,b_ib_j.$$
          Now, by applying of these lemmas we obtain:
          $$sum_i,j=1^nmina_ib_j,a_jb_i-sum_i,j=1^nmina_ia_j,b_ib_j=$$
          $$=sum_i,j=1^nleft(mina_ib_j,a_jb_i-mina_ia_j,b_ib_jright)=sum_i,j=1^nminr_i,r_jx_ix_jgeq0$$ and we are done!



          For $n=4$ we obtain:
          $$(r_4-r_3)x_4^2+(r_3-r_2)(x_3+x_4)^2+(r_2-r_1)(x_2+x_3+x_4)^2+(r_1-r_0)(x_1+x_2+x_3+x_4)^2=$$
          $$=r_1x_1^2+(r_2-r_1+r_1)x_2^2+(r_3-r_2+r_2-r_1+r_1)x_3^2+(r_4-r_3+r_3-r_2+r_2-r_1+r_1)x_4^2+$$
          $$+2r_1x_1x_2+2r_1x_1x_3+2r_1x_1x_4+2(r_2-r_1+r_1)x_2x_3+$$
          $$+2(r_2-r_1+r_1)x_2x_4+2(r_3-r_2+r_2-r_1+r_1)x_3x_4=$$
          $$=r_1x_1^2+r_2x_2^2+x_3r_3^2+r_4x_4^2+2r_1x_1x_2+2r_1x_1x_3+2r_1x_1x_4+$$
          $$+2r_2x_2x_3+2r_2x_2x_4+2r_3x_3x_4.$$






          share|cite|improve this answer






















          • Have you used any telescopic property of series to achieve equality $sum_i=1^nr_ix_i^2+2sum_1leq i<jleq nr_ix_ix_j=sum_i=1^nleft(r_i-r_i-1right)left(sum_j=i^nx_jright)^2$?
            – MathOverview
            Sep 10 at 20:56










          • @MathOverview It's just telescoping summation and $left(sumlimits_i=1^kx_iright)^2=sumlimits_i=1^kx_i^2+2sumlimits_1leq i<jleq kx_ix_j.$
            – Michael Rozenberg
            Sep 10 at 20:58











          • I know that telescopic property for series was somehow used. But I can not understand the equality between these two quantities. Could you explain better?
            – MathOverview
            Sep 10 at 20:59










          • @MathOverview For example, the coefficient before $x_k^2$ it's $sumlimits_i=1^k(r_i-r_i-1)=r_k-r_0=r_k.$ Similarly, the coefficient before $x_ix_j$, where $i<j$ it's $2r_i$.
            – Michael Rozenberg
            Sep 10 at 21:07











          • @MichaelRozenberg can you please elaborate on the step pointed out earlier by MathOverview
            – saisanjeev
            Sep 11 at 14:08












          up vote
          5
          down vote










          up vote
          5
          down vote









          It's USA 2000. The following solution is not mine.



          Since if $mina_i,b_i=0$ then $mina_ia_j,b_ib_j=mina_ib_j,a_jb_i=0,$



          we can assume that $a_i>0$ and $b_i>0$.



          Lemma 1.



          Let $r_igeq0$ and $x_i$ be real numbers. Prove that:
          $$sum_i,j=1^nminr_i,r_jx_ix_jgeq0.$$
          Proof.



          Since $minr_i,r_j=minr_j,r_i,$ we can assume that $0=r_0leq r_1leq r_2leq...leq r_n$ and we obtain:
          $$sum_i,j=1^nminr_i,r_jx_ix_j=sum_i=1^nr_ix_i^2+2sum_1leq i<jleq nr_ix_ix_j=sum_i=1^nleft(r_i-r_i-1right)left(sum_j=i^nx_jright)^2geq0.$$
          Lemma 2.



          Let $a_i>0$, $b_i>0$, $r_i=fracmaxa_i,b_imina_i,b_i-1$ and $x_i=sign(a_i-b_i)mina_i,b_i.$ Prove that:
          $$mina_ib_j,a_jb_i-mina_ia_j,b_ib_j=minr_i,r_jx_ix_j.$$
          Proof.



          Since replacing $a_i$ with $b_i$ gives replacing signs of the both sides, we can assume that $a_igeq b_i$.



          Similarly, we can assume that $a_jgeq b_j$ and we obtain:
          $$minr_i,r_jx_ix_j=minleftfraca_ib_i-1,fraca_jb_j-1rightb_ib_j=$$
          $$=mina_ib_j-b_ib_j,a_jb_i-b_ib_j=mina_ib_j,a_jb_i-b_ib_j=$$
          $$=mina_ib_j,a_jb_i-mina_ia_j,b_ib_j.$$
          Now, by applying of these lemmas we obtain:
          $$sum_i,j=1^nmina_ib_j,a_jb_i-sum_i,j=1^nmina_ia_j,b_ib_j=$$
          $$=sum_i,j=1^nleft(mina_ib_j,a_jb_i-mina_ia_j,b_ib_jright)=sum_i,j=1^nminr_i,r_jx_ix_jgeq0$$ and we are done!



          For $n=4$ we obtain:
          $$(r_4-r_3)x_4^2+(r_3-r_2)(x_3+x_4)^2+(r_2-r_1)(x_2+x_3+x_4)^2+(r_1-r_0)(x_1+x_2+x_3+x_4)^2=$$
          $$=r_1x_1^2+(r_2-r_1+r_1)x_2^2+(r_3-r_2+r_2-r_1+r_1)x_3^2+(r_4-r_3+r_3-r_2+r_2-r_1+r_1)x_4^2+$$
          $$+2r_1x_1x_2+2r_1x_1x_3+2r_1x_1x_4+2(r_2-r_1+r_1)x_2x_3+$$
          $$+2(r_2-r_1+r_1)x_2x_4+2(r_3-r_2+r_2-r_1+r_1)x_3x_4=$$
          $$=r_1x_1^2+r_2x_2^2+x_3r_3^2+r_4x_4^2+2r_1x_1x_2+2r_1x_1x_3+2r_1x_1x_4+$$
          $$+2r_2x_2x_3+2r_2x_2x_4+2r_3x_3x_4.$$






          share|cite|improve this answer














          It's USA 2000. The following solution is not mine.



          Since if $mina_i,b_i=0$ then $mina_ia_j,b_ib_j=mina_ib_j,a_jb_i=0,$



          we can assume that $a_i>0$ and $b_i>0$.



          Lemma 1.



          Let $r_igeq0$ and $x_i$ be real numbers. Prove that:
          $$sum_i,j=1^nminr_i,r_jx_ix_jgeq0.$$
          Proof.



          Since $minr_i,r_j=minr_j,r_i,$ we can assume that $0=r_0leq r_1leq r_2leq...leq r_n$ and we obtain:
          $$sum_i,j=1^nminr_i,r_jx_ix_j=sum_i=1^nr_ix_i^2+2sum_1leq i<jleq nr_ix_ix_j=sum_i=1^nleft(r_i-r_i-1right)left(sum_j=i^nx_jright)^2geq0.$$
          Lemma 2.



          Let $a_i>0$, $b_i>0$, $r_i=fracmaxa_i,b_imina_i,b_i-1$ and $x_i=sign(a_i-b_i)mina_i,b_i.$ Prove that:
          $$mina_ib_j,a_jb_i-mina_ia_j,b_ib_j=minr_i,r_jx_ix_j.$$
          Proof.



          Since replacing $a_i$ with $b_i$ gives replacing signs of the both sides, we can assume that $a_igeq b_i$.



          Similarly, we can assume that $a_jgeq b_j$ and we obtain:
          $$minr_i,r_jx_ix_j=minleftfraca_ib_i-1,fraca_jb_j-1rightb_ib_j=$$
          $$=mina_ib_j-b_ib_j,a_jb_i-b_ib_j=mina_ib_j,a_jb_i-b_ib_j=$$
          $$=mina_ib_j,a_jb_i-mina_ia_j,b_ib_j.$$
          Now, by applying of these lemmas we obtain:
          $$sum_i,j=1^nmina_ib_j,a_jb_i-sum_i,j=1^nmina_ia_j,b_ib_j=$$
          $$=sum_i,j=1^nleft(mina_ib_j,a_jb_i-mina_ia_j,b_ib_jright)=sum_i,j=1^nminr_i,r_jx_ix_jgeq0$$ and we are done!



          For $n=4$ we obtain:
          $$(r_4-r_3)x_4^2+(r_3-r_2)(x_3+x_4)^2+(r_2-r_1)(x_2+x_3+x_4)^2+(r_1-r_0)(x_1+x_2+x_3+x_4)^2=$$
          $$=r_1x_1^2+(r_2-r_1+r_1)x_2^2+(r_3-r_2+r_2-r_1+r_1)x_3^2+(r_4-r_3+r_3-r_2+r_2-r_1+r_1)x_4^2+$$
          $$+2r_1x_1x_2+2r_1x_1x_3+2r_1x_1x_4+2(r_2-r_1+r_1)x_2x_3+$$
          $$+2(r_2-r_1+r_1)x_2x_4+2(r_3-r_2+r_2-r_1+r_1)x_3x_4=$$
          $$=r_1x_1^2+r_2x_2^2+x_3r_3^2+r_4x_4^2+2r_1x_1x_2+2r_1x_1x_3+2r_1x_1x_4+$$
          $$+2r_2x_2x_3+2r_2x_2x_4+2r_3x_3x_4.$$







          share|cite|improve this answer














          share|cite|improve this answer



          share|cite|improve this answer








          edited Sep 11 at 14:28

























          answered Sep 10 at 20:41









          Michael Rozenberg

          89.3k1583179




          89.3k1583179











          • Have you used any telescopic property of series to achieve equality $sum_i=1^nr_ix_i^2+2sum_1leq i<jleq nr_ix_ix_j=sum_i=1^nleft(r_i-r_i-1right)left(sum_j=i^nx_jright)^2$?
            – MathOverview
            Sep 10 at 20:56










          • @MathOverview It's just telescoping summation and $left(sumlimits_i=1^kx_iright)^2=sumlimits_i=1^kx_i^2+2sumlimits_1leq i<jleq kx_ix_j.$
            – Michael Rozenberg
            Sep 10 at 20:58











          • I know that telescopic property for series was somehow used. But I can not understand the equality between these two quantities. Could you explain better?
            – MathOverview
            Sep 10 at 20:59










          • @MathOverview For example, the coefficient before $x_k^2$ it's $sumlimits_i=1^k(r_i-r_i-1)=r_k-r_0=r_k.$ Similarly, the coefficient before $x_ix_j$, where $i<j$ it's $2r_i$.
            – Michael Rozenberg
            Sep 10 at 21:07











          • @MichaelRozenberg can you please elaborate on the step pointed out earlier by MathOverview
            – saisanjeev
            Sep 11 at 14:08
















          • Have you used any telescopic property of series to achieve equality $sum_i=1^nr_ix_i^2+2sum_1leq i<jleq nr_ix_ix_j=sum_i=1^nleft(r_i-r_i-1right)left(sum_j=i^nx_jright)^2$?
            – MathOverview
            Sep 10 at 20:56










          • @MathOverview It's just telescoping summation and $left(sumlimits_i=1^kx_iright)^2=sumlimits_i=1^kx_i^2+2sumlimits_1leq i<jleq kx_ix_j.$
            – Michael Rozenberg
            Sep 10 at 20:58











          • I know that telescopic property for series was somehow used. But I can not understand the equality between these two quantities. Could you explain better?
            – MathOverview
            Sep 10 at 20:59










          • @MathOverview For example, the coefficient before $x_k^2$ it's $sumlimits_i=1^k(r_i-r_i-1)=r_k-r_0=r_k.$ Similarly, the coefficient before $x_ix_j$, where $i<j$ it's $2r_i$.
            – Michael Rozenberg
            Sep 10 at 21:07











          • @MichaelRozenberg can you please elaborate on the step pointed out earlier by MathOverview
            – saisanjeev
            Sep 11 at 14:08















          Have you used any telescopic property of series to achieve equality $sum_i=1^nr_ix_i^2+2sum_1leq i<jleq nr_ix_ix_j=sum_i=1^nleft(r_i-r_i-1right)left(sum_j=i^nx_jright)^2$?
          – MathOverview
          Sep 10 at 20:56




          Have you used any telescopic property of series to achieve equality $sum_i=1^nr_ix_i^2+2sum_1leq i<jleq nr_ix_ix_j=sum_i=1^nleft(r_i-r_i-1right)left(sum_j=i^nx_jright)^2$?
          – MathOverview
          Sep 10 at 20:56












          @MathOverview It's just telescoping summation and $left(sumlimits_i=1^kx_iright)^2=sumlimits_i=1^kx_i^2+2sumlimits_1leq i<jleq kx_ix_j.$
          – Michael Rozenberg
          Sep 10 at 20:58





          @MathOverview It's just telescoping summation and $left(sumlimits_i=1^kx_iright)^2=sumlimits_i=1^kx_i^2+2sumlimits_1leq i<jleq kx_ix_j.$
          – Michael Rozenberg
          Sep 10 at 20:58













          I know that telescopic property for series was somehow used. But I can not understand the equality between these two quantities. Could you explain better?
          – MathOverview
          Sep 10 at 20:59




          I know that telescopic property for series was somehow used. But I can not understand the equality between these two quantities. Could you explain better?
          – MathOverview
          Sep 10 at 20:59












          @MathOverview For example, the coefficient before $x_k^2$ it's $sumlimits_i=1^k(r_i-r_i-1)=r_k-r_0=r_k.$ Similarly, the coefficient before $x_ix_j$, where $i<j$ it's $2r_i$.
          – Michael Rozenberg
          Sep 10 at 21:07





          @MathOverview For example, the coefficient before $x_k^2$ it's $sumlimits_i=1^k(r_i-r_i-1)=r_k-r_0=r_k.$ Similarly, the coefficient before $x_ix_j$, where $i<j$ it's $2r_i$.
          – Michael Rozenberg
          Sep 10 at 21:07













          @MichaelRozenberg can you please elaborate on the step pointed out earlier by MathOverview
          – saisanjeev
          Sep 11 at 14:08




          @MichaelRozenberg can you please elaborate on the step pointed out earlier by MathOverview
          – saisanjeev
          Sep 11 at 14:08

















           

          draft saved


          draft discarded















































           


          draft saved


          draft discarded














          StackExchange.ready(
          function ()
          StackExchange.openid.initPostLogin('.new-post-login', 'https%3a%2f%2fmath.stackexchange.com%2fquestions%2f2911918%2fhow-to-prove-this-inequality-involving-the-minimum-of-two-chosen-numbers%23new-answer', 'question_page');

          );

          Post as a guest













































































          這個網誌中的熱門文章

          Why am i infinitely getting the same tweet with the Twitter Search API?

          Is there any way to eliminate the singular point to solve this integral by hand or by approximations?

          Strongly p-embedded subgroups and p-Sylow subgroups.